Logical Reasoning Question -- help!? Forum

Prepare for the LSAT or discuss it with others in this forum.
Post Reply
aptorres

New
Posts: 19
Joined: Wed Oct 07, 2009 8:29 pm

Logical Reasoning Question -- help!?

Post by aptorres » Thu Feb 11, 2010 7:30 pm

Maria won this year's local sailboat race by beating Sue, the winner in each of the four previous years. We can conclude from this that Maria trained hard.

The conclusion follows logically if which one of the following is assumed?

a. Sue did not train as hard as Maria trained.
b. If Maria trained hard, she would win the sailboat race.
c. Maria could beat a four-time winner only if she trained hard.
d. If Sue trained hard, she would win the sailboat race.
e. Sue is usually a faster sailboat racer than Maria.

The correct answer is 'C'. I chose answer 'B'. Can someone please explain the difference between these two answers?

User avatar
FreeGuy

Bronze
Posts: 130
Joined: Sun Dec 21, 2008 4:42 pm

Re: Logical Reasoning Question -- help!?

Post by FreeGuy » Thu Feb 11, 2010 7:37 pm

B is a mistaken reversal (confusing necessary and sufficient).

C is a restatement of the arg.

"only if" is a necessary indicator.

aptorres

New
Posts: 19
Joined: Wed Oct 07, 2009 8:29 pm

Re: Logical Reasoning Question -- help!?

Post by aptorres » Thu Feb 11, 2010 7:43 pm

Thanks. Doesn't the sufficient condition, i.e. answer B, result in the same answer tho?

User avatar
ConMan345

Silver
Posts: 577
Joined: Tue Sep 29, 2009 1:08 pm

Re: Logical Reasoning Question -- help!?

Post by ConMan345 » Thu Feb 11, 2010 7:50 pm

aptorres wrote:Thanks. Doesn't the sufficient condition, i.e. answer B, result in the same answer tho?
Assuming B, she would win if she trained hard, but she could have won without training hard, too.

ilovethelsat

New
Posts: 33
Joined: Tue Feb 09, 2010 3:03 am

Re: Logical Reasoning Question -- help!?

Post by ilovethelsat » Thu Feb 11, 2010 7:50 pm

aptorres wrote:Thanks. Doesn't the sufficient condition, i.e. answer B, result in the same answer tho?
No. Answer B just tells you that hard work was sufficient to allow Maria to win. It doesn't tell you whether other actions would have been sufficient for Maria to win. (For instance, maybe losing weight or getting more sleep would have been sufficient for Maria to win.)

aptorres

New
Posts: 19
Joined: Wed Oct 07, 2009 8:29 pm

Re: Logical Reasoning Question -- help!?

Post by aptorres » Thu Feb 11, 2010 7:57 pm

I see the light. (thanks to both of you).

Do any of you two do private tutoring in the DC-metro area by any chance? :)

Want to continue reading?

Register now to search topics and post comments!

Absolutely FREE!


Post Reply

Return to “LSAT Prep and Discussion Forum”